Finding the definite integral $int_1^e frac{dx}{xsqrt{1+ln^2x}}$











up vote
5
down vote

favorite
2












So I have the following problem:



$$int_1^{e} frac{1}{xsqrt{1+ln^2x}}dx $$



Can somebody comfirm that the integral of this is



$$ln|sqrt{1+ln^2x}+ ln x|+C$$



and I that the anwser is $$ln |sqrt{2}+1|$$
that is aproximately 0.8814



Does anyone else got the same anwser?










share|cite|improve this question









New contributor




Student123 is a new contributor to this site. Take care in asking for clarification, commenting, and answering.
Check out our Code of Conduct.
















  • 3




    Note that the derivative and the integral are inverse operations of each other... at least up to a constant. If you have the antiderivative, you simply can take the derivative of the antiderivative to check if it is correct.
    – Decaf-Math
    2 days ago






  • 1




    You did an indefinite integral. You would have to plug in 1 and e as bounds to get what you actually want.
    – AHusain
    2 days ago










  • Differentiate your ans, with respect to x.
    – John Nash
    2 days ago















up vote
5
down vote

favorite
2












So I have the following problem:



$$int_1^{e} frac{1}{xsqrt{1+ln^2x}}dx $$



Can somebody comfirm that the integral of this is



$$ln|sqrt{1+ln^2x}+ ln x|+C$$



and I that the anwser is $$ln |sqrt{2}+1|$$
that is aproximately 0.8814



Does anyone else got the same anwser?










share|cite|improve this question









New contributor




Student123 is a new contributor to this site. Take care in asking for clarification, commenting, and answering.
Check out our Code of Conduct.
















  • 3




    Note that the derivative and the integral are inverse operations of each other... at least up to a constant. If you have the antiderivative, you simply can take the derivative of the antiderivative to check if it is correct.
    – Decaf-Math
    2 days ago






  • 1




    You did an indefinite integral. You would have to plug in 1 and e as bounds to get what you actually want.
    – AHusain
    2 days ago










  • Differentiate your ans, with respect to x.
    – John Nash
    2 days ago













up vote
5
down vote

favorite
2









up vote
5
down vote

favorite
2






2





So I have the following problem:



$$int_1^{e} frac{1}{xsqrt{1+ln^2x}}dx $$



Can somebody comfirm that the integral of this is



$$ln|sqrt{1+ln^2x}+ ln x|+C$$



and I that the anwser is $$ln |sqrt{2}+1|$$
that is aproximately 0.8814



Does anyone else got the same anwser?










share|cite|improve this question









New contributor




Student123 is a new contributor to this site. Take care in asking for clarification, commenting, and answering.
Check out our Code of Conduct.











So I have the following problem:



$$int_1^{e} frac{1}{xsqrt{1+ln^2x}}dx $$



Can somebody comfirm that the integral of this is



$$ln|sqrt{1+ln^2x}+ ln x|+C$$



and I that the anwser is $$ln |sqrt{2}+1|$$
that is aproximately 0.8814



Does anyone else got the same anwser?







calculus integration definite-integrals closed-form






share|cite|improve this question









New contributor




Student123 is a new contributor to this site. Take care in asking for clarification, commenting, and answering.
Check out our Code of Conduct.











share|cite|improve this question









New contributor




Student123 is a new contributor to this site. Take care in asking for clarification, commenting, and answering.
Check out our Code of Conduct.









share|cite|improve this question




share|cite|improve this question








edited 17 hours ago





















New contributor




Student123 is a new contributor to this site. Take care in asking for clarification, commenting, and answering.
Check out our Code of Conduct.









asked 2 days ago









Student123

313




313




New contributor




Student123 is a new contributor to this site. Take care in asking for clarification, commenting, and answering.
Check out our Code of Conduct.





New contributor





Student123 is a new contributor to this site. Take care in asking for clarification, commenting, and answering.
Check out our Code of Conduct.






Student123 is a new contributor to this site. Take care in asking for clarification, commenting, and answering.
Check out our Code of Conduct.








  • 3




    Note that the derivative and the integral are inverse operations of each other... at least up to a constant. If you have the antiderivative, you simply can take the derivative of the antiderivative to check if it is correct.
    – Decaf-Math
    2 days ago






  • 1




    You did an indefinite integral. You would have to plug in 1 and e as bounds to get what you actually want.
    – AHusain
    2 days ago










  • Differentiate your ans, with respect to x.
    – John Nash
    2 days ago














  • 3




    Note that the derivative and the integral are inverse operations of each other... at least up to a constant. If you have the antiderivative, you simply can take the derivative of the antiderivative to check if it is correct.
    – Decaf-Math
    2 days ago






  • 1




    You did an indefinite integral. You would have to plug in 1 and e as bounds to get what you actually want.
    – AHusain
    2 days ago










  • Differentiate your ans, with respect to x.
    – John Nash
    2 days ago








3




3




Note that the derivative and the integral are inverse operations of each other... at least up to a constant. If you have the antiderivative, you simply can take the derivative of the antiderivative to check if it is correct.
– Decaf-Math
2 days ago




Note that the derivative and the integral are inverse operations of each other... at least up to a constant. If you have the antiderivative, you simply can take the derivative of the antiderivative to check if it is correct.
– Decaf-Math
2 days ago




1




1




You did an indefinite integral. You would have to plug in 1 and e as bounds to get what you actually want.
– AHusain
2 days ago




You did an indefinite integral. You would have to plug in 1 and e as bounds to get what you actually want.
– AHusain
2 days ago












Differentiate your ans, with respect to x.
– John Nash
2 days ago




Differentiate your ans, with respect to x.
– John Nash
2 days ago










2 Answers
2






active

oldest

votes

















up vote
5
down vote













Hint. One may perform the change of variable
$$
u=ln x,qquad du=frac{dx}x,
$$
giving
$$
int frac{1}{xsqrt{1+ln^2x}}:dx=int frac{du}{sqrt{1+u^2}}
$$
then one may notice that
$$
left[ln left(u+ sqrt{1+u^2}right)right]'=frac{1+frac{u}{sqrt{1+u^2}}}{u+sqrt{1+u^2}}=frac{1}{sqrt{1+u^2}}.
$$






share|cite|improve this answer





















  • thank you so much for helping! but i have a question: before you change the variable there is a 'x' in front of the square root, where does it dissapear after changing the variable?
    – Student123
    19 hours ago










  • @Student123 You are welcome! The '$x$' in front of the square root came under the '$dx$' to produce $displaystyle frac{dx}{x}$ which is equal to $du$.
    – Olivier Oloa
    18 hours ago




















up vote
2
down vote













once we preform the change of variables $u=log x$, we of course have
$$I=intfrac{mathrm{d}u}{sqrt{1+u^2}}$$
Which can be computed using the following identity with hyperbolic trig. functions:
$$cosh^2t-sinh^2t=1$$
Substitution, baby: $u=sinh tRightarrow mathrm{d}u=cosh t mathrm{d}t$
$$I=intfrac{cosh t mathrm{d}t}{sqrt{1+sinh^2t}}$$
$$I=intfrac{cosh t mathrm{d}t}{sqrt{cosh^2t}}$$
$$I=intfrac{cosh t mathrm{d}t}{cosh t}$$
$$I=intmathrm{d}t$$
$$I=t$$
$$I=text{arcsinh},u$$
$$I=text{arcsinh},log x$$
Noting that $$text{arcsinh},x=logbig(sqrt{x^2+1}+xbig)$$
Of course provides the integral:
$$I=logbigg(sqrt{log^2x+1}+log xbigg)$$
QED



Remember that $log x$ is the natural logarithm.



Edit:



adding the absolute value bars in like so:
$$I=logbigg|sqrt{log^2x+1}+log xbigg|$$
Extends the domain of the anti-derivative, which is useful if required.






share|cite|improve this answer























    Your Answer





    StackExchange.ifUsing("editor", function () {
    return StackExchange.using("mathjaxEditing", function () {
    StackExchange.MarkdownEditor.creationCallbacks.add(function (editor, postfix) {
    StackExchange.mathjaxEditing.prepareWmdForMathJax(editor, postfix, [["$", "$"], ["\\(","\\)"]]);
    });
    });
    }, "mathjax-editing");

    StackExchange.ready(function() {
    var channelOptions = {
    tags: "".split(" "),
    id: "69"
    };
    initTagRenderer("".split(" "), "".split(" "), channelOptions);

    StackExchange.using("externalEditor", function() {
    // Have to fire editor after snippets, if snippets enabled
    if (StackExchange.settings.snippets.snippetsEnabled) {
    StackExchange.using("snippets", function() {
    createEditor();
    });
    }
    else {
    createEditor();
    }
    });

    function createEditor() {
    StackExchange.prepareEditor({
    heartbeatType: 'answer',
    convertImagesToLinks: true,
    noModals: true,
    showLowRepImageUploadWarning: true,
    reputationToPostImages: 10,
    bindNavPrevention: true,
    postfix: "",
    imageUploader: {
    brandingHtml: "Powered by u003ca class="icon-imgur-white" href="https://imgur.com/"u003eu003c/au003e",
    contentPolicyHtml: "User contributions licensed under u003ca href="https://creativecommons.org/licenses/by-sa/3.0/"u003ecc by-sa 3.0 with attribution requiredu003c/au003e u003ca href="https://stackoverflow.com/legal/content-policy"u003e(content policy)u003c/au003e",
    allowUrls: true
    },
    noCode: true, onDemand: true,
    discardSelector: ".discard-answer"
    ,immediatelyShowMarkdownHelp:true
    });


    }
    });






    Student123 is a new contributor. Be nice, and check out our Code of Conduct.










     

    draft saved


    draft discarded


















    StackExchange.ready(
    function () {
    StackExchange.openid.initPostLogin('.new-post-login', 'https%3a%2f%2fmath.stackexchange.com%2fquestions%2f3003007%2ffinding-the-definite-integral-int-1e-fracdxx-sqrt1-ln2x%23new-answer', 'question_page');
    }
    );

    Post as a guest















    Required, but never shown

























    2 Answers
    2






    active

    oldest

    votes








    2 Answers
    2






    active

    oldest

    votes









    active

    oldest

    votes






    active

    oldest

    votes








    up vote
    5
    down vote













    Hint. One may perform the change of variable
    $$
    u=ln x,qquad du=frac{dx}x,
    $$
    giving
    $$
    int frac{1}{xsqrt{1+ln^2x}}:dx=int frac{du}{sqrt{1+u^2}}
    $$
    then one may notice that
    $$
    left[ln left(u+ sqrt{1+u^2}right)right]'=frac{1+frac{u}{sqrt{1+u^2}}}{u+sqrt{1+u^2}}=frac{1}{sqrt{1+u^2}}.
    $$






    share|cite|improve this answer





















    • thank you so much for helping! but i have a question: before you change the variable there is a 'x' in front of the square root, where does it dissapear after changing the variable?
      – Student123
      19 hours ago










    • @Student123 You are welcome! The '$x$' in front of the square root came under the '$dx$' to produce $displaystyle frac{dx}{x}$ which is equal to $du$.
      – Olivier Oloa
      18 hours ago

















    up vote
    5
    down vote













    Hint. One may perform the change of variable
    $$
    u=ln x,qquad du=frac{dx}x,
    $$
    giving
    $$
    int frac{1}{xsqrt{1+ln^2x}}:dx=int frac{du}{sqrt{1+u^2}}
    $$
    then one may notice that
    $$
    left[ln left(u+ sqrt{1+u^2}right)right]'=frac{1+frac{u}{sqrt{1+u^2}}}{u+sqrt{1+u^2}}=frac{1}{sqrt{1+u^2}}.
    $$






    share|cite|improve this answer





















    • thank you so much for helping! but i have a question: before you change the variable there is a 'x' in front of the square root, where does it dissapear after changing the variable?
      – Student123
      19 hours ago










    • @Student123 You are welcome! The '$x$' in front of the square root came under the '$dx$' to produce $displaystyle frac{dx}{x}$ which is equal to $du$.
      – Olivier Oloa
      18 hours ago















    up vote
    5
    down vote










    up vote
    5
    down vote









    Hint. One may perform the change of variable
    $$
    u=ln x,qquad du=frac{dx}x,
    $$
    giving
    $$
    int frac{1}{xsqrt{1+ln^2x}}:dx=int frac{du}{sqrt{1+u^2}}
    $$
    then one may notice that
    $$
    left[ln left(u+ sqrt{1+u^2}right)right]'=frac{1+frac{u}{sqrt{1+u^2}}}{u+sqrt{1+u^2}}=frac{1}{sqrt{1+u^2}}.
    $$






    share|cite|improve this answer












    Hint. One may perform the change of variable
    $$
    u=ln x,qquad du=frac{dx}x,
    $$
    giving
    $$
    int frac{1}{xsqrt{1+ln^2x}}:dx=int frac{du}{sqrt{1+u^2}}
    $$
    then one may notice that
    $$
    left[ln left(u+ sqrt{1+u^2}right)right]'=frac{1+frac{u}{sqrt{1+u^2}}}{u+sqrt{1+u^2}}=frac{1}{sqrt{1+u^2}}.
    $$







    share|cite|improve this answer












    share|cite|improve this answer



    share|cite|improve this answer










    answered 2 days ago









    Olivier Oloa

    107k17175293




    107k17175293












    • thank you so much for helping! but i have a question: before you change the variable there is a 'x' in front of the square root, where does it dissapear after changing the variable?
      – Student123
      19 hours ago










    • @Student123 You are welcome! The '$x$' in front of the square root came under the '$dx$' to produce $displaystyle frac{dx}{x}$ which is equal to $du$.
      – Olivier Oloa
      18 hours ago




















    • thank you so much for helping! but i have a question: before you change the variable there is a 'x' in front of the square root, where does it dissapear after changing the variable?
      – Student123
      19 hours ago










    • @Student123 You are welcome! The '$x$' in front of the square root came under the '$dx$' to produce $displaystyle frac{dx}{x}$ which is equal to $du$.
      – Olivier Oloa
      18 hours ago


















    thank you so much for helping! but i have a question: before you change the variable there is a 'x' in front of the square root, where does it dissapear after changing the variable?
    – Student123
    19 hours ago




    thank you so much for helping! but i have a question: before you change the variable there is a 'x' in front of the square root, where does it dissapear after changing the variable?
    – Student123
    19 hours ago












    @Student123 You are welcome! The '$x$' in front of the square root came under the '$dx$' to produce $displaystyle frac{dx}{x}$ which is equal to $du$.
    – Olivier Oloa
    18 hours ago






    @Student123 You are welcome! The '$x$' in front of the square root came under the '$dx$' to produce $displaystyle frac{dx}{x}$ which is equal to $du$.
    – Olivier Oloa
    18 hours ago












    up vote
    2
    down vote













    once we preform the change of variables $u=log x$, we of course have
    $$I=intfrac{mathrm{d}u}{sqrt{1+u^2}}$$
    Which can be computed using the following identity with hyperbolic trig. functions:
    $$cosh^2t-sinh^2t=1$$
    Substitution, baby: $u=sinh tRightarrow mathrm{d}u=cosh t mathrm{d}t$
    $$I=intfrac{cosh t mathrm{d}t}{sqrt{1+sinh^2t}}$$
    $$I=intfrac{cosh t mathrm{d}t}{sqrt{cosh^2t}}$$
    $$I=intfrac{cosh t mathrm{d}t}{cosh t}$$
    $$I=intmathrm{d}t$$
    $$I=t$$
    $$I=text{arcsinh},u$$
    $$I=text{arcsinh},log x$$
    Noting that $$text{arcsinh},x=logbig(sqrt{x^2+1}+xbig)$$
    Of course provides the integral:
    $$I=logbigg(sqrt{log^2x+1}+log xbigg)$$
    QED



    Remember that $log x$ is the natural logarithm.



    Edit:



    adding the absolute value bars in like so:
    $$I=logbigg|sqrt{log^2x+1}+log xbigg|$$
    Extends the domain of the anti-derivative, which is useful if required.






    share|cite|improve this answer



























      up vote
      2
      down vote













      once we preform the change of variables $u=log x$, we of course have
      $$I=intfrac{mathrm{d}u}{sqrt{1+u^2}}$$
      Which can be computed using the following identity with hyperbolic trig. functions:
      $$cosh^2t-sinh^2t=1$$
      Substitution, baby: $u=sinh tRightarrow mathrm{d}u=cosh t mathrm{d}t$
      $$I=intfrac{cosh t mathrm{d}t}{sqrt{1+sinh^2t}}$$
      $$I=intfrac{cosh t mathrm{d}t}{sqrt{cosh^2t}}$$
      $$I=intfrac{cosh t mathrm{d}t}{cosh t}$$
      $$I=intmathrm{d}t$$
      $$I=t$$
      $$I=text{arcsinh},u$$
      $$I=text{arcsinh},log x$$
      Noting that $$text{arcsinh},x=logbig(sqrt{x^2+1}+xbig)$$
      Of course provides the integral:
      $$I=logbigg(sqrt{log^2x+1}+log xbigg)$$
      QED



      Remember that $log x$ is the natural logarithm.



      Edit:



      adding the absolute value bars in like so:
      $$I=logbigg|sqrt{log^2x+1}+log xbigg|$$
      Extends the domain of the anti-derivative, which is useful if required.






      share|cite|improve this answer

























        up vote
        2
        down vote










        up vote
        2
        down vote









        once we preform the change of variables $u=log x$, we of course have
        $$I=intfrac{mathrm{d}u}{sqrt{1+u^2}}$$
        Which can be computed using the following identity with hyperbolic trig. functions:
        $$cosh^2t-sinh^2t=1$$
        Substitution, baby: $u=sinh tRightarrow mathrm{d}u=cosh t mathrm{d}t$
        $$I=intfrac{cosh t mathrm{d}t}{sqrt{1+sinh^2t}}$$
        $$I=intfrac{cosh t mathrm{d}t}{sqrt{cosh^2t}}$$
        $$I=intfrac{cosh t mathrm{d}t}{cosh t}$$
        $$I=intmathrm{d}t$$
        $$I=t$$
        $$I=text{arcsinh},u$$
        $$I=text{arcsinh},log x$$
        Noting that $$text{arcsinh},x=logbig(sqrt{x^2+1}+xbig)$$
        Of course provides the integral:
        $$I=logbigg(sqrt{log^2x+1}+log xbigg)$$
        QED



        Remember that $log x$ is the natural logarithm.



        Edit:



        adding the absolute value bars in like so:
        $$I=logbigg|sqrt{log^2x+1}+log xbigg|$$
        Extends the domain of the anti-derivative, which is useful if required.






        share|cite|improve this answer














        once we preform the change of variables $u=log x$, we of course have
        $$I=intfrac{mathrm{d}u}{sqrt{1+u^2}}$$
        Which can be computed using the following identity with hyperbolic trig. functions:
        $$cosh^2t-sinh^2t=1$$
        Substitution, baby: $u=sinh tRightarrow mathrm{d}u=cosh t mathrm{d}t$
        $$I=intfrac{cosh t mathrm{d}t}{sqrt{1+sinh^2t}}$$
        $$I=intfrac{cosh t mathrm{d}t}{sqrt{cosh^2t}}$$
        $$I=intfrac{cosh t mathrm{d}t}{cosh t}$$
        $$I=intmathrm{d}t$$
        $$I=t$$
        $$I=text{arcsinh},u$$
        $$I=text{arcsinh},log x$$
        Noting that $$text{arcsinh},x=logbig(sqrt{x^2+1}+xbig)$$
        Of course provides the integral:
        $$I=logbigg(sqrt{log^2x+1}+log xbigg)$$
        QED



        Remember that $log x$ is the natural logarithm.



        Edit:



        adding the absolute value bars in like so:
        $$I=logbigg|sqrt{log^2x+1}+log xbigg|$$
        Extends the domain of the anti-derivative, which is useful if required.







        share|cite|improve this answer














        share|cite|improve this answer



        share|cite|improve this answer








        edited yesterday

























        answered yesterday









        clathratus

        1,729219




        1,729219






















            Student123 is a new contributor. Be nice, and check out our Code of Conduct.










             

            draft saved


            draft discarded


















            Student123 is a new contributor. Be nice, and check out our Code of Conduct.













            Student123 is a new contributor. Be nice, and check out our Code of Conduct.












            Student123 is a new contributor. Be nice, and check out our Code of Conduct.















             


            draft saved


            draft discarded














            StackExchange.ready(
            function () {
            StackExchange.openid.initPostLogin('.new-post-login', 'https%3a%2f%2fmath.stackexchange.com%2fquestions%2f3003007%2ffinding-the-definite-integral-int-1e-fracdxx-sqrt1-ln2x%23new-answer', 'question_page');
            }
            );

            Post as a guest















            Required, but never shown





















































            Required, but never shown














            Required, but never shown












            Required, but never shown







            Required, but never shown

































            Required, but never shown














            Required, but never shown












            Required, but never shown







            Required, but never shown







            Popular posts from this blog

            Can a sorcerer learn a 5th-level spell early by creating spell slots using the Font of Magic feature?

            ts Property 'filter' does not exist on type '{}'

            mat-slide-toggle shouldn't change it's state when I click cancel in confirmation window